What is the solution

What Is The Solution

Answers

Answer 1

Answer:

x = -72

Step-by-step explanation:

-72 + 8 = -64

The third square root of -64 is -4


Related Questions

Ariel has 20 bouncy balls. He later then sold them for a price of $2 per ball to his best friend, Mike Hawk. Later, Ariel has gotten back 1/69 of his original amount of bouncy balls and got 20 more. How many bouncy balls does Ariel have?

Answers

ariel has 40 balls left

Answer:

420

Step-by-step explanation:

Hey Guys! Can You Please Help Me In My Math​

Answers

This would be a great tree diagram
Sorry...

pic is not clear.....

pls help me in these questions ​

Answers

Answer:

1=85

2=10

3=108

Step-by-step explanation:

Number 1: Calculate each angle...and you know a straight line is 180°. N the interior sum of a quadrilateral is always 360°.

Number2: Use corresponding, alternate and interior angles method. It will help

Number 3: It's just about solving the interior sum of the pentagon

Answer:

2. 10 degrees

3. 108 degrees

Step-by-step explanation:

sorry I reposted it because the previous answer was deleted and plz mark me as a brainliest.

álgebra 1 solve -7 + 18(17h + 19)

Answers

Answer:  =360h+335

Step-by-step explanation:

Answer:

306h + 281

Step-by-step explanation:

-7 + 18(17h + 19)

-7 + 306h + 288

306h + 281

helpppppppppppppppppp

Answers

Answer:

Dude

sheesh i only seeing 5 and 18
The answer is 5 it is the only number that can be multiplied by another number to get 10

if sam has 20 watermelons and he eats 14 how many does he have left?

Answers

Answer:

6 duh!

you can do this by yourself lol

Which best describes the vertex of the graph?


a (-3, -4)
b (-3, -4)
c (3, -4)
d (3, -4)

Answers

Answer: C

Step-by-step explanation:

C. Is the answer for sure

Which graph represents the solution of x2 + 9y2 ≤ 81 and y2 + 2 < x? On a coordinate plane, an ellipse has center (0, 0) and goes through (3, 0), (0, negative 9), (negative 3, 0), and (0, 9). A parabola opens to the right and goes through (6, 2), has vertex (2, 0), and goes through (6, negative 2). Everything inside of the ellipse and outside of the parabola is shaded. On a coordinate plane, an ellipse has center (0, 0) and goes through (3, 0), (0, negative 9), (negative 3, 0), and (0, 9). A parabola opens to the right and goes through (6, 2), has vertex (2, 0), and goes through (6, negative 2). Everything inside of the ellipse and inside of the parabola is shaded. On a coordinate plane, an ellipse has center (0, 0) and goes through (9, 0), (0, negative 3), (negative 9, 0) and (0, 3). A parabola opens to the right and goes through (6, 2), has vertex (2, 0), and goes through (6, negative 2). Everything inside of the ellipse and inside of the parabola is shaded. On a coordinate plane, an ellipse has center (0, 0) and goes through (9, 0), (0, negative 3), (negative 9, 0) and (0, 3). A parabola opens to the right and goes through (6, 2), has vertex (2, 0), and goes through (6, negative 2). Everything inside of the ellipse and outside of the parabola is shaded.

Answers

9514 1404 393

Answer:

  C

Step-by-step explanation:

The ellipse y-intercepts are ±3, the x-intercepts are ±9, eliminating choices A and B. The parabola is shaded inside, eliminating choice D.

The correct choice is the third one.

Word problem One of the citizens has 97 silver coins. How many bronze coins would it take to equal this amount

Answers

Given: Given that a citizen have 97 silver coins.

To find : Here we need to find that how many bronze coins would it take to equal this amount.

Solution: We know, 1 silver coin=10 bronze coin

So, 97 silver coin=10×97 bronze coin

=970 bronze coin

Therefore, 970 bronze coins would it take to equal this amount.

Someone tell me where everyone is going right please !!

Answers

Answer:

1min = 0.25miles

5.25miles / 0.25miles = 25 = 25minutes

Step-by-step explanation:

Hope this is right I'm not the best at worded time/distance math questions.

9514 1404 393

Answer:

  0 ≤ t < 52.5 minutes

Step-by-step explanation:

Riko will be behind Yuto until her distance traveled matches his. That is, she will be behind for ...

  0.35t < 5.25 +0.25t

  0.10t < 5.25

  t < 52.5

Riko will be behind Yuto on the interval 0 ≤ t < 52.5 minutes.

_____

Additional comment

distance = speed × time

Here, time is measured from when Riko starts riding. In addition to the 5.25 miles that Yuto has already gone, his distance will be the product of his speed (0.25 mi/min) and the travel time (t min). Then Yuto's total distance is 5.25+0.25t miles. The speed×time product is also used to find Riko's distance traveled. In her case, it is 0.35t miles.

Surface area of a cuboid is 384
work out the volume

Answers

Answer:

is 512

Step-by-step explanation:

Find the measure of the missing angle using exterior angle sum theorm

Answers

35 degrees.

Assuming 145 is the exterior angle, the interior is 35. 180-145=35
Add 110 to 35 and get 145 degrees. Which leads the remaining angle 35 degrees

Answer:

35

Step-by-step explanation:

The exterior sum theorem states the exterior angle is equal to the sum of the opposite interior angles

145 = ?+110

145 - 100 = ?

35 = ?

Find the equation of the line through point (2,2) and parallel to y=x+4. Use a forward slash (i.e.”/“) for fractions (e.g. 1/2 for

Answers

Answer:

The equation of the line is, y = x

Step-by-step explanation:

The constraints of the required linear equation are;

The point through which the line passes = (2, 2)

The line to which the required line is parallel = y = x + 4

Two lines are parallel if they have the same slope, therefore, we have;

The slope of the line, y = x + 4 is m = 1

Therefore, the slope of the required line = 1

The equation of the required lime in point and slope form becomes;

y - 2 = 1 × (x - 2)

∴ y = x - 2 + 2 = x

The equation of the required line is therefore, y = x

You invest $4.000 in a savings account. The account pays 3% annual interest. How much money will be in the savings account after 9 years?

A) $4,938.29
B) $5,219.20
C) $5,124.33
D) $6,003.45​

Answers

Answer:

4000*0.03*9=1080

4000+1080=5080

umm but like thats not one of the answers sorry

Hope This Helps!!!

Step-by-step explanation:

4000*3% = 120

120*9= 1080

1080+4000=5080

BRAINLIEST FOR ANSWER WITH EXPLANATION
JUST ONE QUESTION ATTACHED BELOW:

Answers

Answer:

m=16

Step-by-step explanation:

(6+6)/m=6/8

12/m=6/8

M=16

Answer:

16

Step-by-step explanation:

This triangle has been dialated; ILM is a dialated version of IJK. We know that dialations are by scale factors, and we also know the length of two corresponding sides.

IM is 6 m long, and IMK is 6+6 m long, so 12 m. So using simple ratios, we know the dialation factor from ILM to IJK is 2. The corresponding side for JK (m) is LM, which we know is 8 m. So side m is 16 because of the scale factor (2).

Maybe you thought we were solving for "m", as in a varriable all the sides are multiplied by, but I think that's just bad labeling. Hope this helps!

I forgot to label the triangle below! I just know that the cos A = 0.48. Based on this information, which angle should be marked A?

Answers

Answer:

angle 1

Step-by-step explanation:

Using the trigonometric mnemonic SOH CAH TOA, we know that cos or cosine is the ratio between the adjacent side and hypotenuse side.

This means that if cos A = 0.48, A is the measure of the angle which it's relative adjacent side divided by the hypotenuse of the triangle will be around 0.48.

Let's try angle 2, cos (angle 2) = adjacent / hypotenuse = 7.8 / 8.9 = 0.876404494382 ≈ 0.87 ≠ 0.48. Since the proportions are not equal, this angle cannot be the one marked as A.

Since angle 3 is a right angle, the adjacent could be either side so it cannot be correct. Thus angle 1 is correct.

Write the equation of the graph y=

Answers

9514 1404 393

Answer:

  y = 6^x -3

Step-by-step explanation:

The graph is that of an exponential function that has been translated downward. We notice the horizontal asymptote is -3, and a couple of points on the graph are (0, -2) and (1, 3).

The shifted parent function will look like ...

  y = a·b^x +c

where c is the horizontal asymptote. Using the two points we found, we have ...

  -2 = a·b^0 -3 . . . . . using (x, y) = (0, -2)

  1 = a . . . . . . . . . . add 3 and simplify

Then using (x, y) = (1, 3), we have ...

  3 = b^1 -3

  6 = b . . . . . . . . . add 3 and simplify

So, the equation is ...

  y = 6^x -3

Instructions: Problem 2 ! Find the missing angle in the image below. Do not include spaces in your answers

Answers

Step-by-step explanation:

since angles in a triangle add up to 180

<vuw=180-(71+23)

=86°

since angles in a straight line add up to 180

<vuf=180-86

=94

30 points if helped, question in the picture

Answers

Step-by-step explanation:

[tex] \frac{2}{2 + \sqrt{3} } \times \frac{2 - \sqrt{3} }{2 - \sqrt{3} } = \frac{2(2 - \sqrt{3} )}{1} = 4 - 2 \sqrt{3} [/tex]

If the quadratic formula is used to find the solution set of 3x + 4x-2 = 0, what are the solutions? ​

Answers

Answer is c
hope it helps I’m new at this

a train leaves Westchester at 6:30. What time should it arrive at Middlewich

Answers

Answer:

a). 6:51

b). 6:30 am

c). 34 minutes

Step-by-step explanation:

a). Train leaves Westchester at 6:30.

   From the arrival - departure table in column (2),

   Arrival time of the train at middlewich = 6:51

b). Kate has to reach Southam before 9:00 am

   Therefore, time of the latest train that she can catch to get to work on time is 6:30 am

    By this train she can reach at 07:19 at Southam.

c). Duration of journey from Westchester to Eastwick = 06:34 - 06:00

                                                                                        = 00:34

                                                                                        ≈ 34 minutes

Plis help me it’s for today

Answers

Answer:

Following are the solution to the given points:

Step-by-step explanation:

For question 1:

[tex]\to 3^{-4}= \frac{1}{3^4}=\frac{1}{81}=0.0123456789[/tex]

For question 2:

[tex]\to (-2)^{3}\cdot(-2)^{4}\cdot(-2)^{-1}=-8\cdot-16\cdot -\frac{1}{2}= 128\cdot -\frac{1}{2}=-64[/tex]

For question 3:

[tex]\to 7^{-4} \div 7^{-2}= \frac{1}{7^{4}} \div \frac{1}{7^{2}}=\frac{1}{7^{4}} \times \frac{7^{2}}{1}=\frac{1}{7^{2}} =\frac{1}{49} =0.0204081633[/tex]

For question 4:

[tex]\to [(-3)^{2}]^3= (-3)^{2\cdot 3}= (-3)^{6}=729[/tex]

For question 5:

[tex]\to [5 \cdot (-3)]^{2}= 25 \cdot 9=225[/tex]

For question 6:

[tex]\to [(10 \div 5)]^{3}= [(\frac{10}{5})]^{3}=[2]^{3}=8[/tex]

For question 7:

[tex]\to 10^6 \cdot 10^{-4} \cdot 10^2= 10^6 \cdot \frac{1}{10^{4}} \cdot 10^2= 10^2 \cdot 10^2=10^4=10,000[/tex]

For question 8:

[tex]\to (-4)^{-5}=\frac{1}{(-4)^{5}}=- \frac{1}{1,024}=-0.0009765625[/tex]

For question 9:

[tex]\to \frac{2^3}{2^4}= \frac{8}{16}=\frac{1}{2}=0.5[/tex]

For question 10:

[tex]\to (-6)^3 \cdot (-6)^5 \cdot (-6)^{-5}= (-6)^3 \cdot (-6)^5 \cdot \frac{1}{(-6)^{5}}= (-6)^3 =-216[/tex]

Solve for x

Marking brainliest

Answers

Answer:

110°

Step-by-step explanation:

someone please help me ASAP!

Answers

Answer:

214°

Step-by-step explanation:

The measure of an arc that sees the center angle of the circle is equal to the very same angle that it sees

Since the measure of circle is 360° and arc AB is given as 146° the measure of arc ACB should be 360 - 146 = 214°

Please help :(((((((((((((((((((((((((

Answers

jeez that looks really hard

Step-by-step explanation:

can someone answer plssss gives 100 pints i think bc I picked 100

Answers

[tex]1. \frac{20}{100} [/tex]

[tex]2. \frac{1}{5} [/tex]

Answer:  1

Explanation:

There are a few ways to do this. One way is to notice that the jump from 5 to 100 is "times 20" (go from right to left across the bottom denominators).

So we must do the same "times 20" type of jump when going across the numerators. If x is the numerator for the right hand side, then we go from x to 20. That must mean x = 1

Put another way, we could have these steps

20/100 = x/5

20*5 = 100*x ... cross multiplication

100 = 100x

100x = 100

x = 100/100 .... dividing both sides by 100

x = 1

We see that the fraction 20/100 reduces fully to 1/5

To go from 1/5 to 20/100, we multiply both parts by 20 (divide both parts by 20 to go in reverse).

please tel me answer of under root 3+4i
without calculatot with steps

Answers

Hello,

[tex]Let's\ say \\\\z=\sqrt{3+4*i} =a+b*i\\\\z^2=3+4*i=(a+b*i)^2=a^2-b^2+2i*a*b\\\\\\if \ a\neq 0\\\left\{\begin{array}{ccc}a^2+b^2&=&3\\2ab=4\\\end{array}\right.\\\\\\\left\{\begin{array}{ccc}b=\dfrac{2}{a}\\a^2-(\dfrac{2}{a})^2=2\\\end{array}\right.\\\\\\a^4-4=3*a^2\\a^4-3a^2-4=0\\\\\Delta=(-3)^2-4*1*(-4)=25=5^2\\\\a^2=4\ or \ a^2=-1 (impossible)\\\\So:\\(a=2\ and\ b=1)\ or\ (a=-2\ and\ b=-1)\\[/tex]

Roots are thus 2+i and -2-i

There is an other using a geometrical formula (formule de Moivre)

One number is 6 times a second number. The sum of the two numbers is 56. Find the numbersOne number is 6 times a second number.

The sum of the two numbers is 56. Find the numbers

Answers

Answer:

I think its 56 = 56

Step-by-step explanation:

Answer:

8 and 48

Step-by-step explanation:

This can be written as a system of equations. X will be the smaller number and y will be the larger:

6x = y

x + y = 56

Then substitute y in the second equation for 6x(seen in first equation) and solve.

x + 6x = 56

7x = 56

x = 8

So one number is 8. To find the other plug in 8 for x in either of the original equations (both get the same answer for y).

6x = y

6 * 8 = y

48 = y

Then double check by seeing if they fit the requirements of the problem.

Hope this helps!

The number of clicks for a search text ad is 50 and the number of impressions is 5000. The CTR would be Group of answer choices 1% 2% 5% 10%

Answers

Given:

Clicks = 50

Impressions = 5000

To find:

The CTR percentage.

Solution:

We know that,

[tex]CTR=\dfrac{\text{Clicks}}{\text{Impressions}}\times 100[/tex]

Substituting the given values, we get

[tex]CTR=\dfrac{50}{5000}\times 100[/tex]

[tex]CTR=\dfrac{1}{100}\times 100[/tex]

[tex]CTR=1\%[/tex]

Therefore, the correct option is A.

please help!
factor f(x)=6x^2-x-2​

Answers

Hi there!

[tex]\large\boxed{(3x - 2)(2x + 1) }[/tex]

6x² - x - 2

To solve, we must factor into the following format:

(ax - b)(cx - d)

The following conditions must be met:

a · c = 6

da + bd = -1

b · d = -2

By guessing and checking, we get:

(3x - 2)(2x + 1)

Answer for Acellus:

(2 x + 1) (3 x - 2)

Step-by-step explanation:

i checked the equation on an online calculator, and it came up with it switched (compared to the other answer that is posted) like so...

I put the answer in and it said it was right for Acellus.

Hope this helps :))

Other Questions
the representative particle for KBr is the? Identify the structure you are consuming when you eat each of the following vegetables. Match the words in the left column to the appropriate blanks in the sentences on the right. a. stem b. taprootc. lateral buds d. modified steme. leaf (petiole and blade)f. petiole 1. Asparagus 2. Brussels sprouts 3. Celery 4. Spinach 5. Carrot 6. Potato = O EQUATIONS AND INEQUALITIES Solving a decimal word problem using a linear equation of the... Jayden Esp Leila received a $90 gift card for a coffee store. She used it in buying some coffee that cost $7.26 per pound. After buying the coffee, she had $60.96 left on her card. How many pounds of coffee did she buy? Suppose 42% of the population has myopia. If a random sample of size 442 is selected, what is the probability that the proportion of persons with myopia will differ from the population proportion by less than 3% What governmental principle of the United States is based on Montesquieu's concerns?A. federalismB. republicanismC. freedom of speechD. separation of powers A _____________ is a small farm in which farmers share what they produce as well as their profit.a.sovkhozc.kolkhozb.chernobyld.perestroika PLISSSSSS HELP!!!!!!!!!!!!! i will give brainliest..... Read the text and answer the questions a.Which planets did voyager I visit ayudaaaaa no se que hay que hacer Color blindness is a recessive sex-linked human trait. If a color-blind father and a mother with normal color vision have a color-blind son, what must be the mother's genotype?A. XNXnB. XNXNC. XnYnD. XNY Help! Can you please help please answer this A delivery boy on a bicycle drags a wagon full of newspapers by pedaling at 0.90 m/s for 45 minutes using a force of 40 N. How much work has the boy done? Una muestra de S2 (g) se coloca en un recipiente rgido vaco a 800 K y sta ejerce una presin inicial de 0,92 atm, luego se combina para formar S8 (g) mediante la reaccin: S2 (g) S8 (g) Una vez alcanzado el equilibrio, la presin parcial de S2 (g) ha disminuido a 0,18 atm. Calcule Kp para la reaccin a esta temperatura Choose the statement that best describes Galarzas figurative language in this passage. PLEASE HELP ASAP!!!!!!!! The Western Front was characterized by A.the slow but steady advance of the German army.B. trench warfare that kept both sides in virtually the same positions for four years. C. decisive victories by the French army, quickly driving back the German forces. D.innovative strategies and tactics that fully utilized the new technologies available to both armies. need help on question 6 How do you think humans adaptability will continue to help our species survive in the future?(Please not from the internet in your own words) -5y-9=-(y-1) equation a -1/2b -2 1/2c -2d -2/5 A petting zoo gives out large buckets of healthy popcorn snacks for visitors to feed to the animals. what fraction of the snack is left in the bucket after a cow eats 1/3, a goose eats 1/12, and a goat eats 2/9?